Fix Thre -> Three typo in Serway & Jewett v8's problem 23.9.
authorW. Trevor King <wking@drexel.edu>
Tue, 3 Apr 2012 13:47:35 +0000 (09:47 -0400)
committerW. Trevor King <wking@drexel.edu>
Tue, 3 Apr 2012 13:47:35 +0000 (09:47 -0400)
latex/problems/Serway_and_Jewett_8/problem23.09.tex

index 3f63004920d8d9e8a41ce35b75abe2cd8bf2bfd9..66f6da162aca49f0b8d98a450bee4a530ab5f2cd 100644 (file)
@@ -1,5 +1,5 @@
 \begin{problem*}{23.9}
 \begin{problem*}{23.9}
-Thre point charges are arranged as shown in Figure~P23.9.
+Three point charges are arranged as shown in Figure~P23.9.
 Find \Part{a} the magnitude and \Part{b} the direction of the electric
 force on the particle at the origin.
 % y5.00nC      6.00nC
 Find \Part{a} the magnitude and \Part{b} the direction of the electric
 force on the particle at the origin.
 % y5.00nC      6.00nC